Help me please.
Whoever answers right gets brainliest

Help Me Please.Whoever Answers Right Gets Brainliest

Answers

Answer 1

Answer:y=x-5

Step-by-step explanation:this is the answer for sure

Answer 2

Answer:

[tex]y=x-5[/tex]

Step-by-step explanation:

Notice for every X you do increase your Y by 1, this is the slope must be 1, to check you can use the slope equation with any two points

[tex]m= \frac{-4-(-3)}{1-(2)} =-1[/tex]
The lets use one point and the line equation:

[tex]y-yo=m(x-xo)[/tex]
[tex]y+4=-1(x-1)[/tex]

Solving for Y you can obtain

[tex]y=x-5[/tex]

You can input any X value and check the Y value to be sure, for example, for X=3 you can check Y=3-5=-2. In the table for X=3 the Y value is -2, so this checks.


Related Questions

4.1 h(x) Consider h(c) = cos 2x 4.1.1 Complete the table below, rounding your answer off to the first decimal where needed: -90° -75° -60° -45° -30° -15° 0° 15° 30° 45° 60° 75° 90° 4.1.2 Now use the table and draw the graph of h(x) = cos 2x on the system of axes below: -90°-75°-60-45-30-15 2- 14 - 1+ -24 (2) 15° 30° 45° 60⁰ 75⁰ 90⁰ (2) (2)​

Answers

Here's the completed table and the graph:

x  h(x)

-90°  1.0

-75°  -0.5

-60°  -1.0

-45° -0.0

-30° 1.0

-15° 0.5

0° 1.0

15° 0.5

30° -0.0

45° -1.0

60° -0.5

75° 1.0

90° 1.0

What is function?

In mathematics, a function is a relation between a set of inputs and a set of possible outputs, with the property that each input is related to exactly one output. Functions are often represented as a set of ordered pairs, where the first element of each pair is an input and the second element is the corresponding output. Functions are a fundamental concept in many areas of mathematics and have many real-world applications, including in science, engineering, and economics.

Here,

To calculate the values of h(c) in the table, we plug in the given values of x into the function h(c) = cos 2x and evaluate. For example, to find h(c) when x = -75°:

h(c) = cos 2x

h(c) = cos 2(-75°) (substitute -75° for x)

h(c) = cos (-150°) (simplify using the double angle identity)

h(c) = -0.5 (evaluate using the unit circle or a calculator)

We repeat this process for each value of x to fill out the table.

To graph the function h(x) = cos 2x, we plot each point from the table on the given system of axes. The x-axis represents the angle x in degrees, and the y-axis represents the value of h(x) = cos 2x. We then connect the points with a smooth curve to obtain the graph.

To know more about function,

https://brainly.com/question/28278699

#SPJ1

the figure below shows the change of a population over time. which statement best describes the mode of selection depicted in the figure?

Answers

The statement that best describes the mode of selection depicted in the figure is (b) Directional Selection, changing the average color of population over time.

The Directional selection is a type of natural selection that occurs when individuals with a certain trait or phenotype are more likely to survive and reproduce than individuals with other traits or phenotypes.

In the directional selection of evolution, the mean shifts that means average shifts to one extreme and supports one trait and leads to eventually removal of the other trait.

In this case, one end of the extreme-phenotypes which means that the dark-brown rats are being selected for. So, over the time, the average color of the rat population will change.

Therefore, the correct option is (b).

Learn more about Directional Selection here

https://brainly.com/question/12226774

#SPJ4

The given question is incomplete, the complete question is

The figure below shows the change of a population over time. which statement best describes the mode of selection depicted in the figure?

(a) Disruptive Official, favoring the average individual

(b) Directional Selection, changing the average color of population over time

(c) Directional selection, favoring the average individual

(d) Stabilizing Selection, changing the average color of population over time

Mr. wings class collected empty soda, cans for recycling project. each of the 20 students had to collect 40 cans. Each can has a mass of 15 grams. How many kilograms of cans did the class collect to recycle?

A 0.6 kg.
B 12 kg
C 12,000 kg
D 12,000,000 kg

Answers

Step-by-step explanation:

40 cans/student   X   20 students  X  15 gram/can = 12 000 gm  = 12 kg

The Venn diagram here shows the cardinality of each set. Use this to find the cardinality of the given set.
n(A)=

Answers

The cardinality of set A, n(A) = 29

What is cardinality of a set?

The cardinality of a set is the total number of elements in the set

Given the Venn diagram here shows the cardinality of each set. To find the cardinality of set A, n(A), we proceed as follows.

Since the cardinality of a set is the total number of elements in the set, then cardinality of set A , n(A) = 9 + 8 + 3 + 9

= 29

So, n(A) = 29

Learn more about cardinality of a set here:

https://brainly.com/question/27846845

#SPJ1

Equation of the line in the graph is y=? X + ?

Answers

to get the equation of any straight line, we simply need two points off of it, let's use those two in the picture below

[tex](\stackrel{x_1}{-3}~,~\stackrel{y_1}{2})\qquad (\stackrel{x_2}{3}~,~\stackrel{y_2}{-4}) \\\\\\ \stackrel{slope}{m}\implies \cfrac{\stackrel{\textit{\large rise}} {\stackrel{y_2}{-4}-\stackrel{y1}{2}}}{\underset{\textit{\large run}} {\underset{x_2}{3}-\underset{x_1}{(-3)}}} \implies \cfrac{-6}{3 +3} \implies \cfrac{ -6 }{ 6 } \implies - 1[/tex]

[tex]\begin{array}{|c|ll} \cline{1-1} \textit{point-slope form}\\ \cline{1-1} \\ y-y_1=m(x-x_1) \\\\ \cline{1-1} \end{array}\implies y-\stackrel{y_1}{2}=\stackrel{m}{- 1}(x-\stackrel{x_1}{(-3)}) \implies y -2 = - 1 ( x +3) \\\\\\ y-2=-x-3\implies {\Large \begin{array}{llll} y=-x-1 \end{array}}[/tex]

Maximize z = 3x₁ + 5x₂
subject to: x₁ - 5x₂ ≤ 35
3x1 - 4x₂ ≤21
with. X₁ ≥ 0, X₂ ≥ 0.
use simplex method to solve it and find the maximum value​

Answers

Answer:

See below.

Step-by-step explanation:

We can solve this linear programming problem using the simplex method. We will start by converting the problem into standard form

Maximize z = 3x₁ + 5x₂ + 0s₁ + 0s₂

subject to

x₁ - 5x₂ + s₁ = 35

3x₁ - 4x₂ + s₂ = 21

x₁, x₂, s₁, s₂ ≥ 0

Next, we create the initial tableau

Basis x₁ x₂ s₁ s₂ RHS

s₁ 1 -5 1 0 35

s₂ 3 -4 0 1 21

z -3 -5 0 0 0

We can see that the initial basic variables are s₁ and s₂. We will use the simplex method to find the optimal solution.

Step 1: Choose the most negative coefficient in the bottom row as the pivot element. In this case, it is -5 in the x₂ column.

Basis x₁ x₂ s₁ s₂ RHS

s₁ 1 -5 1 0 35

s₂ 3 -4 0 1 21

z -3 -5 0 0 0

Step 2: Find the row in which the pivot element creates a positive quotient when each element in that row is divided by the pivot element. In this case, we need to find the minimum positive quotient of (35/5) and (21/4). The minimum is (21/4), so we use the second row as the pivot row.

Basis x₁ x₂ s₁ s₂ RHS

s₁ 4/5 0 1/5 1 28/5

x₂ -3/4 1 0 -1/4 -21/4

z 39/4 0 15/4 3/4 105

Step 3: Use row operations to create zeros in the x₂ column.

Basis x₁ x₂ s₁ s₂ RHS

s₁ 1 0 1/4 7/20 49/10

x₂ 0 1 3/16 -1/16 -21/16

z 0 0 39/4 21/4 525/4

The optimal solution is x₁ = 49/10, x₂ = 21/16, and z = 525/4.

Therefore, the maximum value of z is 525/4, which occurs when x₁ = 49/10 and x₂ = 21/16.

Question 6 of 10
Based only on the information given in the diagram, which congruence
theorems or postulates could be given as reasons why ACDE AOPQ?
Check all that apply.
AA
A. AAS
B. ASA
C. LL
OD. HL
E. LA
F. SAS

Answers

Therefore, A, B, C, and F are the proper responses as the congruence theories or postulates based on the data.

what is triangle ?

Having three straight sides and three angles where they intersect, a triangle is a closed, two-dimensional shape. It is one of the fundamental geometric shapes and has a number of characteristics that can be used to study and resolve issues that pertain to it. The triangle inequality theory states that the sum of a triangle's interior angles is always 180 degrees, and that the longest side is always the side across from the largest angle. Triangles can be used to solve a wide range of mathematical issues in a variety of disciplines and can be categorised based on the length of their sides and the measurement of their angles.

given

We can use the following congruence theories or postulates based on the data in the diagram:

A. ASA

B. AAS

C. LL (corresponding angles hypothesis)

F. SAS

Therefore, A, B, C, and F are the proper responses as the congruence theories or postulates based on the data.

To know more about triangle visit:

https://brainly.com/question/2773823

#SPJ1

Can someone please help with these 4

Answers

Answer:

Step-by-step explanation:

1)  b      (acute is less than 90)

2)  a     (obtuse: more than 90, less than 180)

3)  c

4) c

Answer:

1. NOM, JOK, KOL

2. MOL, NOK, MOJ

3. NOJ, JOL

4. NOL, MOK

PLEASE HELP MEE with all four questionsss

Answers

Therefore, the distance between the 90 degree angle and the hypotenuse is approximately 0.829 units.

What is triangle?

A triangle is a two-dimensional geometric shape that is formed by three straight line segments that connect to form three angles. It is one of the most basic shapes in geometry and has a wide range of applications in mathematics, science, engineering, and everyday life. Triangles can be classified by the length of their sides (equilateral, isosceles, or scalene) and by the size of their angles (acute, right, or obtuse). The study of triangles is an important part of geometry, and their properties and relationships are used in many areas of mathematics and science.

Here,

1. To find HF, we can use the angle bisector theorem, which states that if a line bisects an angle of a triangle, then it divides the opposite side into two segments that are proportional to the adjacent sides. Let's denote the length of HF as x. Then, by the angle bisector theorem, we have:

JF/FH = JG/HG

Substituting the given values, we get:

15/x = 18/21

Simplifying and solving for x, we get:

x = 15 * 21 / 18

x = 17.5

Therefore, HF is 17.5 cm.

2. Let's denote the length of the hypotenuse as h and the length of the leg opposite the 18-unit perpendicular as a. We can then use the Pythagorean theorem to write:

h² = a²  + 18²

We are told that the hypotenuse is divided into segments of length x and 6 units, so we can write:

h = x + 6

Substituting this expression into the first equation, we get:

(x + 6)² = a² + 18²

We are also told that the leg adjacent to the angle opposite the 4-unit segment is divided into segments of length 4 and (a - 4), so we can write:

a = 4 + (a - 4)

Simplifying this equation, we get:

a = a

Now we can substitute this expression for a into the previous equation and solve for x:

(x + 6)² = (4 + (a - 4))² + 18²

Expanding and simplifying, we get:

x² + 12x - 36 = 0

Using the quadratic formula, we get:

x = (-12 ± √(12² - 4(1)(-36))) / (2(1))

x = (-12 ± √(288)) / 2

x = -6 ± 6√(2)

Since the length of a segment cannot be negative, we take the positive root:

x = -6 + 6sqrt(2)

x ≈ 1.46

Therefore, the value of x is approximately 1.46 units.

3. Let's denote the length of the hypotenuse as h and the length of the leg adjacent to the angle opposite the 9-unit perpendicular as b. We can then use the Pythagorean theorem to write:

h² = b² + 9²

We are told that the hypotenuse is divided into segments of length x and 6 units, so we can write:

h = x + 6

Substituting this expression into the first equation, we get:

(x + 6)² = b² + 9²

Expanding and simplifying, we get:

x² + 12x - b² = 27

We also know that the length of the leg opposite the 9-unit perpendicular is:

a = √(h² - 9²)

= √((x + 6)² - 9²)

= √(x² + 12x + 27)

Now we can use the fact that the tangent of the angle opposite the 9-unit perpendicular is equal to the ratio of the lengths of the opposite and adjacent sides:

tan(θ) = a / b

Substituting the expressions for a and b, we get:

tan(θ) = √(x² + 12x + 27) / (x + 6)

We also know that the tangent of the angle theta is equal to the ratio of the length of the opposite side to the length of the adjacent side:

tan(θ) = 9 / b

Substituting the expression for b, we get:

tan(θ) = 9 / √(h² - 9²)

Substituting the expression for h, we get:

tan(θ) = 9 / √((x + 6)² - 9²)

Since the tangent function is the same for equal angles, we can set these two expressions for the tangent equal to each other:

√(x² + 12x + 27) / (x + 6) = 9 / √((x + 6)² - 9²)

Squaring both sides, we get:

(x² + 12x + 27) / (x + 6)² = 81 / ((x + 6)² - 81)

Cross-multiplying and simplifying, we get:

x⁴ + 36x³ + 297x² - 1458x - 2916 = 0

Using a numerical method such as the Newton-Raphson method or the bisection method, we can find the approximate solution to this equation:

x ≈ 9.449

Therefore, the value of x is approximately 9.449 units.

4. Let's denote the length of the hypotenuse as h and the length of the leg adjacent to the angle opposite the distance we want to find as b. We can use the Pythagorean theorem to write:

h² = b² + d²

We are told that the hypotenuse is divided into segments of length 9 and 4 units, so we can write:

h = 9 + 4 = 13

Substituting this expression into the first equation, we get:

13² = b² + d²

Simplifying and solving for d, we get:

d = √(13² - b²)

Now, we need to find the value of b. We know that the hypotenuse is divided into segments of length 9 and 4 units, so we can use similar triangles to write:

b / 4 = 9 / 13

Simplifying and solving for b, we get:

b = 36 / 13

Substituting this expression for b into the equation we found earlier for d, we get:

d = √(13² - (36/13)²)

Simplifying and finding a common denominator, we get:

d =√ (169*13 - 36²) / 13²

Simplifying further, we get:

d = √(169169 - 3636) / 169

Calculating this expression, we get:

d ≈ 0.829

To know more about triangle,

https://brainly.com/question/28600396

#SPJ1

Show your solution ( 3. ) C + 18 = 29

Answers

Answer:

Show your solution ( 3. ) C + 18 = 29

Step-by-step explanation:

To solve the equation C + 18 = 29, we want to isolate the variable C on one side of the equation.

We can start by subtracting 18 from both sides of the equation:

C + 18 - 18 = 29 - 18

Simplifying the left side of the equation:

C = 29 - 18

C = 11

Therefore, the solution to the equation C + 18 = 29 is C = 11.

given a function f(x), find the critical values and use the critical values to find intervals of increasing/deacreasing, maxes and mins.

Answers

The critical values, the intervals of increasing or decreasing and the maximum and minimum points of the f(x) is (-1.5, -16), x < -1.5 and x = -1.5 and for b (4,6) and (2,10), (2,4).

A) Critical values

We will find out the critical value by solving for f ' (x) = 0

therefore, taking the derivative of given function we get,

f ' (x) = 4(2x) + 12 = 0

        = 8x + 12 = 0

therefore, 8x = -12

                 x = -12/8

                 x= -1.5

x = -1.5 is the only critical value in x-coordinate. Now to determine the y-coordinate, simply put the value of x in the function f(x) = 4x2 + 12x - 7

we get, f(-1.5) = 4(-1.5)2 + 12 (-1.5) - 7

                      = 4(2.25) - 18 - 7

                      = 9 - 25 = -16  

therefore, the critical value of the function f(x) = 4x2 + 12x - 7 is (-1.5, -16)

f(x) =x3 - 9x2 + 24x - 10.

Intervals of increasing and decreasing function is i.e. f decreases for

x < -1.5.

Therefore, f has minimum value at x = -1.5.

B) Critical values

We will find out the critical value by solving for f ' (x) = 0

therefore, taking the derivative of given function we get,

f '(x) = 3x2 - 9(2x) + 24

       = 3x2 - 18x + 24 = 0

therefore, 3 ( x2 - 6x + 8) = 0

   i.e x2 - 6x + 8 = 0

        (x-4) (x-2) = 0

So, x = 4 or x = 2 are the two critical values in x-coordinate. Now to determine the y-coordinate, simply put the values of x in the function f(x) =x3 - 9x2 + 24x - 10

we get, Substituting x = 4

f(4) = 43 - 9 (4)2 +24 (4) -10

     = 64 - 144 + 96 - 10

     = 6

Now, Substituting x = 2

f(2) = 23 - 9(2)2 + 24(2) - 10

     = 8 - 36 + 48 - 10

     = 10

Therefore, the critical values of the function f(x) =x3 - 9x2 + 24x - 10 are (4,6) and (2,10).

Intervals of increasing and decreasing functions is f decreases in (2,4).

therefore, f has minimum at x = 4 and maximum at x = 2.

Learn more about Critical values:

https://brainly.com/question/15970126

#SPJ4

Complete question:

For each function determine: i) the critical values ii) the intervals of increasing or decreasing iii) the maximum and minimum points.

a. f(x) = 4x²+12x–7 (3 marks)

b. F(x) = x°-9x²+24x-10 (3 marks)

According to Money magazine, Maryland had the highest median annual household income of any state in 2018 at $75,847.† Assume that annual household income in Maryland follows a normal distribution with a median of $75,847 and standard deviation of $33,800.
(a) What is the probability that a household in Maryland has an annual income of $90,000 or more? (Round your answer to four decimal places.)
(b) What is the probability that a household in Maryland has an annual income of $50,000 or less? (Round your answer to four decimal places.)

Answers

The required probability that a household in Maryland with annual income of ,

$90,000 or more is equal to 0.3377.

$50,000 or less is equal to 0.2218.

Annual household income in Maryland follows a normal distribution ,

Median =  $75,847

Standard deviation = $33,800

Probability of household in Maryland has an annual income of $90,000 or more.

Let X be the random variable representing the annual household income in Maryland.

Then,

find P(X ≥ $90,000).

Standardize the variable X using the formula,

Z = (X - μ) / σ

where μ is the mean (or median, in this case)

And σ is the standard deviation.

Substituting the given values, we get,

Z = (90,000 - 75,847) / 33,800

⇒ Z = 0.4187

Using a standard normal distribution table

greater than 0.4187  as 0.3377.

P(X ≥ $90,000)

= P(Z ≥ 0.4187)

= 0.3377

Probability that a household in Maryland has an annual income of $90,000 or more is 0.3377(rounded to four decimal places).

Probability that a household in Maryland has an annual income of $50,000 or less.

P(X ≤ $50,000).

Standardizing X, we get,

Z = (50,000 - 75,847) / 33,800

⇒ Z = -0.7674

Using a standard normal distribution table

Probability that a standard normal variable is less than -0.7674 as 0.2218. This implies,

P(X ≤ $50,000)

= P(Z ≤ -0.7674)

= 0.2218

Probability that a household in Maryland has an annual income of $50,000 or less is 0.2218.

Therefore, the probability with annual income of $90,000 or more and  $50,000 or less is equal to 0.3377 and 0.2218 respectively.

learn more about probability here

brainly.com/question/24111146

#SPJ4

Answer this imagine please

Answers

The expression that is not equivalent to the model shown is given as follows:

-4(3 + 2). -> Option C.

What are equivalent expressions?

Equivalent expressions are mathematical expressions that have the same value, even though they may look different. In other words, two expressions are equivalent if they produce the same output for any input value.

The expression for this problem is given by three times the subtraction of four, plus three times the addition of 2, hence:

3(-4) + 3(2) = -12 + 6 = 3(-4 + 2) = 3(-2) = -6.

Hence the expression that is not equivalent is the expression given in option C, for which the result is given as follows:

-4(3 + 2) = -4 x 5 = -20.

More can be learned about equivalent expressions at https://brainly.com/question/15775046

#SPJ1

find the following answer

Answers

According to the Venn diagram the value of [tex]n(A ^ C \cap B ^ C) = {3}[/tex] so the number of elements in that set is 1.

What is Venn diagram ?

A Venn diagram is a diagram that shows all possible logical relations between a finite collection of different sets. It is usually represented as a rectangle or a circle for each set and the overlapping areas between them, showing the common elements that belong to more than one set. Venn diagrams are widely used in mathematics, logic, statistics, and computer science to visualize the relationships between different sets and help solve problems related to set theory.

According to the question:
To solve this problem, we first need to understand the notation used.

n(A) denotes the set A and the numbers within the braces {} indicate the elements in set A. For example, n(A)={7,4,3,9} means that the set A contains 7, 4, 3, and 9.

n(AnB) denotes the intersection of sets A and B, i.e., the elements that are common to both A and B. For example, n(AnB)={4,3} means that the sets A and B have 4 and 3 in common.

^ denotes intersection of sets

cap denotes the intersection of sets

Now, we need to find the elements that are common to sets A and C, and sets B and C. We can do this by taking the intersection of A and C, and the intersection of B and C, and then taking the intersection of the two resulting sets.

[tex]n(A ^ C) = n(A \cap C) = {3,9}[/tex]

[tex]n(B ^ C) = n(B\cap C) = {3,5}[/tex]

Now, we take the intersection of [tex]n(A ^ C)[/tex] and [tex]n(B ^ C)[/tex]:

[tex]n(A ^ C \cap B ^ C) = {3}[/tex]

Therefore, the answer is 1.


To know more about Venn diagrams visit:
https://brainly.com/question/28060706
#SPJ1

10.5.PS-18 Question content area top Part 1 The diagram shows a track composed of a rectangle with a semicircle on each end. The area of the rectangle is square meters 11200. What is the perimeter of the​ track? Use 3.14 for pi.

Answers

First, we need to find the length and width of the rectangle:

Let's suppose the width of the rectangle is "w" and the length is "l".

The area of the rectangle is given as 11200 square meters:

lw = 11200

Now, we need to find the radius of each semicircle. Since the semicircle completes the circle with the rectangle width "w", the radius would be half of the width:

r = w/2

The perimeter of the track can be found by adding the perimeter of the rectangle with the circumference of both semicircles:

Perimeter = 2(l + w) + 2πr
Perimeter = 2(l + w) + 2(3.14)(w/2)
Perimeter = 2(l + w) + 3.14w

We know the area of the rectangle, which is lw = 11200, but we need to find the values of l and w. We can do this by trying different values of l and w that multiply to give 11200.

Here are a few possibilities:
l = 280, w = 40 -> Perimeter = 2(280 + 40) + 3.14(40) = 681.6
l = 560, w = 20 -> Perimeter = 2(560 + 20) + 3.14(20) = 1166.8
l = 1120, w = 10 -> Perimeter = 2(1120 + 10) + 3.14(10) = 2253.6

Therefore, the perimeter of the track could be 681.6 meters, 1166.8 meters, or 2253.6 meters, depending on the values of length and width. Without additional information, we cannot determine the exact perimeter of the track

The Venn diagram here shows the cardinality of each set. Use this to find the cardinality of the given set.

Answers

With the help of the given Venn diagram, the answer of n(A∪B) is 44 respectively.

What is the Venn diagram?

A Venn diagram is a visual representation that makes use of circles to highlight the connections between different objects or limited groups of objects.

Circles that overlap share certain characteristics, whereas circles that do not overlap do not.

Venn diagrams are useful for showing how two concepts are related and different visually.

When two or more objects have overlapping attributes, a Venn diagram offers a simple way to illustrate the relationships between them.

Venn diagrams are frequently used in reports and presentations because they make it simpler to visualize data.

So, we need to find:

A ∪ B

Now, calculate as follows:

The collection of all objects found in either the Blue or Green circles, or both, is known as A B. Its components number is:

8 + 7 + 14 + 6 + 1 + 8 = 44

n(A∪B) = 44

Therefore, with the help of the given Venn diagram, the answer of n(A∪B) is 44 respectively.

Know more about the Venn diagram here:

https://brainly.com/question/2099071

#SPJ1

Given that x + 1/2 = 5, what is 2*x^2 - 3x + 6 - 3/x +2/x^2

pls help me soon

Answers

Sure, let's solve this step-by-step:

First, we need to solve for x in the equation x + 1/2 = 5.

We can do this by subtracting 1/2 from both sides, giving us x = 4 1/2.

Now, we can substitute x = 4 1/2 into the equation 2*x^2 - 3x + 6 - 3/x +2/x^2.

We can simplify the equation by multiplying both sides by x^2, giving us:

2*x^2 - 3x + 6 - 3/x +2 = 10*x^2 - 3x + 6.

Now, we can combine all of the terms with x:

10*x^2 - 6x + 6 = 0.

Finally, we can solve the equation using the quadratic formula:

x = 3/5 or x = 2.

Therefore, the answer to the equation is 10*(3/5)^2 - 6(3/5) + 6 = 4.8, or 10*2^2 - 6(2) + 6 = 16.

WILL MARK AS BRAINLIEST!!!!!!!!!!!!!!
If "f" is differentiable and f(1) < f(2), then there is a number "c", in the interval (_____, _____) such that f'(c)>_______

Answers

If "f" is differentiable and f(1) < f(2), then there is a number "c", in the interval  (1, 2)  such that f'(c)>  0.

How do we know?

Applying the  Mean Value Theorem for derivatives, if a function f is continuous on the closed interval [a, b] and differentiable on the open interval (a, b), then there exists at least one number c in the interval (a, b) such that:

f'(c) = (f(b) - f(a)) / (b - a)

In the scenario above, we have that f is differentiable, and that f(1) < f(2).

choosing a = 1 and b = 2.

Then applying the Mean Value Theorem, there exists at least one number c in the interval (1, 2) such that:

f'(c) = (f(2) - f(1)) / (2 - 1)

f'(c) = f(2) - f(1)

We have that f(1) < f(2), we have:

f(2) - f(1) > 0

We can conclude by saying that there exists a number c in the interval (1, 2) such that:

f'(c) = f(2) - f(1) > 0

Learn more about Mean Value Theorem at: https://brainly.com/question/19052862

#SPJ1

Bria is a customer who would like to display her collection of soap carvings on top of her bookcase. The collection needs an area of 300 square inches. What should b equal for the top of the bookcase to have the correct area? Round your answer to the nearest tenth of an inch. I need help
D: Please !!!!​

Answers

Answer:

We can use the formula for the area of a rectangle to solve this problem. Let's assume that the length of the top of the bookcase is L and the width is b. Then, we can write:

L × b = 300

Solving for b, we get:

b = 300 / L

Since we don't know the length L, we cannot find the exact value of b. However, we can use the given information to make an estimate. Let's say that the length of the bookcase is 60 inches. Then, we have:

b = 300 / 60 = 5

So, if the length of the bookcase is 60 inches, the width needs to be at least 5 inches to accommodate Bria's soap carving collection. However, if the length is different, the required width will also be different.

Locate the absolute extrema of the function on the closed interval

Answers

Answer:

The absolute extrema is minimum at (-1, 2/9)

Step-by-step explanation:

Absolute extrema is a logical point that shows whether a the curve function is maximum or minimum.

Forexample a curve in the image attached. A, B and C are points of absolute maxima or absolute maximum. and P and Q are points of absolute minima or minimum.

Remember A, B, C, P, Q are critical points or stationary points.

How do we find absolute extrema?

The find the sign of the second derivative of the function.

From the question;

[tex]{ \sf{g(x) = \sqrt[3]{x} }} \\ \\ { \sf{g(x) = {x}^{ \frac{1}{3} } }} \\ [/tex]

Find the first derivative of g(x)

[tex]{ \sf{g {}^{l}(x) = \frac{1}{3} {x}^{ - \frac{2}{3} } }} \\ [/tex]

Find the second derivative;

[tex]{ \sf{g {}^{ll} (x) = ( \frac{1}{3} \times - \frac{2}{3}) {x}^{( - \frac{2}{3} - 1) } }} \\ \\ { \sf{g {ll}^{(x)} = - \frac{2}{9} {x}^{ - \frac{5}{3} } }}[/tex]

Then substitute for x as -1 from [-1, 1]

[tex]{ \sf{g {}^{ll}(x) = - \frac{2}{9} ( - 1) {}^{ - \frac{5}{3} } }} \\ \\ = \frac{ - 2}{9} \times - 1 \\ \\ = \frac{2}{9} [/tex]

Since the sign of the result is positive, the absolute extrema is minimum

Parts A-D. What is the value of the sample mean as a percent? What is its interpretation? Compute the sample variance and sample standard deviation as a percent as measures of rotelle for the quarterly return for this stock.

Answers

The sample mean is 2.1, the sample variance is 212.5% and the standard deviation is 14.57%

What is the sample mean?

a. The sample mean can be computed as the average of the quarterly percent total returns:

[tex](11.2 - 20.5 + 13.2 + 12.6 + 9.5 - 5.8 - 17.7 + 14.3) / 8 = 2.1[/tex]

So the sample mean is 2.1%, which can be interpreted as the average quarterly percent total return for the stock over the sample period.

b. The sample variance can be computed using the formula:

[tex]s^2 = sum((x - mean)^2) / (n - 1)[/tex]

where x is each quarterly percent total return, mean is the sample mean, and n is the sample size. Plugging in the values, we get:

[tex]s^2 = (11.2 - 2.1)^2 + (-20.5 - 2.1)^2 + (13.2 - 2.1)^2 + (12.6 - 2.1)^2 + (9.5 - 2.1)^2 + (-5.8 - 2.1)^2 + (-17.7 - 2.1)^2 + (14.3 - 2.1)^2 / (8 - 1) = 212.15[/tex]

So the sample variance is 212.15%. The sample standard deviation can be computed as the square root of the sample variance:

[tex]s = \sqrt(s^2) = \sqrt(212.15) = 14.57[/tex]

So the sample standard deviation is 14.57%.

c. To construct a 95% confidence interval for the population variance, we can use the chi-square distribution with degrees of freedom n - 1 = 7. The upper and lower bounds of the confidence interval can be found using the chi-square distribution table or calculator, as follows:

upper bound = (n - 1) * s^2 / chi-square(0.025, n - 1) = 306.05

lower bound = (n - 1) * s^2 / chi-square(0.975, n - 1) = 91.91

So the 95% confidence interval for the population variance is (91.91, 306.05).

d. To construct a 95% confidence interval for the standard deviation (as percent), we can use the formula:

lower bound = s * √((n - 1) / chi-square(0.975, n - 1))

upper bound = s * √((n - 1) / chi-square(0.025, n - 1))

Plugging in the values, we get:

lower bound = 6.4685%

upper bound = 20.1422%

So the 95% confidence interval for the standard deviation (as percent) is (6.4685%, 20.1422%).

Learn more on sample mean here;

https://brainly.com/question/26941429

#SPJ1

x±Z./
x±t./
A highway safety researcher is studying the design of a freeway sign and is interested
in the mean maximum distance at which drivers are able to read the sign. The
maximum distances (in feet) at which a random sample of 9 drivers can read the sign are as follows:
400 600 600 600 650 500 345 500 440
The mean of the sample of 9 distances is 512 feet with a standard deviation of 105
feet.
(a) What assumption must you make before constructing a confidence interval?
•The population distribution is Uniform.
•The population distribution is Normal.
(b) At the 90% confidence level what is the margin of error on your estimate of the true mean maximum distance at which drivers can read the sign.
Answer= feet (round to the nearest whole number)
(c) Construct a 90% confidence interval estimate of the true mean maximum
distance at which drivers can read the sign.
Lower value= feet (round to the nearest whole number)
Upper value= feet (round to the nearest whole number)
(d) There is a 10% chance the error on the estimate is bigger than what value?
Answer= feet (round to the nearest whole number)
(e) The researcher wants to reduce the margin of error to only 15 feet at the 90% confidence level. How many additional drivers need to be sampled? Assume the sample standard deviation is a close estimate of the population standard deviation.
Answer=

Answers

In response to the stated question, we may state that The margin of error function is equal to the highest mistake on the estimate.

what is function?

In mathematics, a function is a connection between two sets of numbers in which each member of the first set (known as the domain) corresponds to a single element in the second set (called the range). In other words, a function takes inputs from one set and produces outputs from another. Inputs are commonly represented by the variable x, whereas outputs are represented by the variable y. A function can be described using an equation or a graph. The equation y = 2x + 1 represents a linear function in which each value of x yields a distinct value of y.v

(a) The population distribution must be assumed to be normal before generating a confidence interval.

(b) The margin of error with 90% confidence is provided by:

Error Margin = Z (/2) * (/n)

Where Z (/2) is the confidence level/2 crucial value, is the population standard deviation (unknown), and n is the sample size.

Error Margin = t (/2, n-1) * (s/n)

Where t (/2, n-1) is the critical value for the degrees of freedom /2 and n-1, and s is the sample standard deviation.

(d) The margin of error is equal to the highest mistake on the estimate.

To know more about function visit:

https://brainly.com/question/28193995

#SPJ1

help I’ll give brainliest ^•^ just question (7) thanks!!

Answers

Answer:

To shift the graph of f(x) = |x| to have a domain of [-3, 6], we need to move the left endpoint from -6 to -3 and the right endpoint from 3 to 6.

A translation to the right by 3 units will move the left endpoint of the graph of f(x) to -3, but it will also shift the right endpoint to 6 + 3 = 9, which is outside the desired domain.

A translation to the left by 3 units will move the right endpoint of the graph of f(x) to 3 - 3 = 0, which is outside the desired domain.

A translation upward or downward will not change the domain of the graph, so options B and D can be eliminated.

Therefore, the correct answer is C g(x) = x - 3. This translation will move the left endpoint to -3 and the right endpoint to 6, which is exactly the desired domain.

If n is an integer and n > 1, then n! is the product of n and every other positive integer that is less than n. for example, 5! = 5 x 4 x 3 x 2 x 1a. Write 6! in standard factored formb. Write 20! in standard factored formc. Without computing the value of (20!)2, determine how many zeros are at the end of this number when it is written in decimal form. Justify your answer

Answers

a. 6! = 6 x 5 x 4 x 3 x 2 x 1 = 720. b. 20! = 20 x 19 x 18 x ... x 2 x 1. c. There are 16 zeros at the end of the decimal representation of (20!)2.

a. 6! = 6 x 5 x 4 x 3 x 2 x 1 = 720

b. 20! = 20 x 19 x 18 x ... x 2 x 1. To write this in factored form, we can identify the prime factors of each number and write the product using exponents. For example, 20 = 2² x 5, so we can write 20! as:

20! = (2² x 5) x 19 x (2 x 3²) x 17 x (2² x 7) x 13 x (2 x 2 x 3) x 11 x (2³) x (3) x (2) x 7 x (2) x 5 x (2) x 3 x 2 x 1

Simplifying, we get:

20! = 2¹⁸ x 3⁸ x 5⁴ x 7² x 11 x 13 x 17 x 19

c. The number of zeros at the end of (20!)² in decimal form is determined by the number of factors of 10, which is equivalent to the number of factors of 2 x 5. Since there are more factors of 2 than 5 in the prime factorization of (20!)², we only need to count the number of factors of 5. There are four factors of 5 in the prime factorization of 20!, which contribute four factors of 10 to the square. Therefore, (20!)²ends in 8 zeros when written in decimal form.

Learn more about integer here: brainly.com/question/15276410

#SPJ4

Find the value of x. If your answer is not an integer, leave it in simplest radical form. The diagram is not drawn to scale.
NOTE: Enter your answer and show all the steps that you use to solve this problem in the space provided. Use the 30°-60°-90° Triangle Theorem to find the answer.

Answers

The value of the x is 5√3 after we successfully do the application of the 30°-60°-90° Triangle theorem.

What is Triangle theorem?

The 30°-60°-90° Triangle Theorem states that in such a triangle, the side opposite the 30° angle is half the length of the hypotenuse, and the side opposite the 60° angle is the product of the length of the hypotenuse and the square root of 3 divided by 2.

Using this theorem, we can write:

y = hypotenuse

Opposite of 30° angle = 5 = hypotenuse/2

Opposite of 60° angle = x = hypotenuse × (√(3)/2)

Solving for the hypotenuse in terms of y from the first equation, we get:

hypotenuse = 5×2 = 10

Substituting this value into the third equation, we get:

x = 10 × (√(3)/2) = 5 × √(3)

To know more about hypotenuse, visit:

https://brainly.com/question/29407794

#SPJ1

CALCULUS HELP NEEDED: Express the integrand as a sum of partial fractions and evaluate the integrals.

[tex]\int\ {\frac{x+3}{2x^{3}-8x}} \, dx[/tex]

**I know I need to solve for A&B, but I have no idea where to start for partial fractions.

Answers

The integral of the function expressed as sum of partial frictions is -3/8 ln|x| + 7/8 ln|x + 2| - 1/8 ln|x - 2| + C.

What is the integral of function?

First, factor out 2x from the denominator to obtain:

∫[(x + 3)/(2x³ - 8x)] dx = ∫[(x + 3)/(2x)(x² - 4)] dx

Next, we use partial fractions to express the integrand as a sum of simpler fractions. To do this, we need to factor the denominator of the integrand:

2x(x² - 4) = 2x(x + 2)(x - 2)

Therefore, we can write:

(x + 3)/(2x)(x² - 4) = A/(2x) + B/(x + 2) + C/(x - 2)

Multiplying both sides by the denominator, we get:

x + 3 = A(x + 2)(x - 2) + B(2x)(x - 2) + C(2x)(x + 2)

Now, we need to find the values of A, B, and C. We can do this by equating coefficients of like terms:

x = A(x² - 4) + B(2x² - 4x) + C(2x² + 4x)

x = (A + 2B + 2C)x² + (-4A - 4B + 4C)x - 4A

Equating coefficients of x², x, and the constant term, respectively, we get:

A + 2B + 2C = 0

-4A - 4B + 4C = 1

-4A = 3

Solving for A, B, and C, we find:

A = -3/4

B = 7/16

C = -1/16

Therefore, the partial fraction decomposition is:

(x + 3)/(2x)(x² - 4) = -3/(4(2x)) + 7/(16(x + 2)) - 1/(16(x - 2))

The integral becomes:

∫[(x + 3)/(2x³ - 8x)] dx = ∫[-3/(8x) + 7/(8(x + 2)) - 1/(8(x - 2))] dx

Integrating each term separately gives:

∫[-3/(8x) + 7/(8(x + 2)) - 1/(8(x - 2))] dx

= -3/8 ln|x| + 7/8 ln|x + 2| - 1/8 ln|x - 2| + C

where;

C is the constant of integration.

Therefore, the final answer is:

∫[(x + 3)/(2x³ - 8x)] dx = -3/8 ln|x| + 7/8 ln|x + 2| - 1/8 ln|x - 2| + C

Learn more about partial fractions here: https://brainly.com/question/24594390

#SPJ1

Show your complete solution

4. 5x-13=12

Answers

Answer: x = 5

Step-by-step explanation:

To solve for x, we can first add 13 to both sides to isolate the variable term:

5x - 13 + 13 = 12 + 13

Simplifying the left side and evaluating the right side:

5x = 25

Then, divide both sides by 5 to isolate x:

5x/5 = 25/5

Simplifying:

x = 5

Therefore, the solution to the equation 5x - 13 = 12 is x = 5.

To solve for x in the equation 5x-13=12, we want to isolate the variable x on one side of the equation. We can do this by adding 13 to both sides of the equation:

5x-13+13 = 12+13

Simplifying, we get:

5x = 25

Finally, we can solve for x by dividing both sides of the equation by 5:

5x/5 = 25/5

Simplifying, we get:

x = 5

Therefore, the solution to the equation 5x-13=12 is x = 5.

When expressions of the form (x −r)(x − s) are multiplied out, a quadratic polynomial is obtained. For instance, (x −2)(x −(−7))= (x −2)(x + 7) = x2 + 5x − 14.
a. What can be said about the coefficients of the polynomial obtained by multiplying out (x −r)(x − s) when both r and s are odd integers? when both r and s are even integers? when one of r and s is even and the other is odd?
b. It follows from part (a) that x2 − 1253x + 255 cannot be written as a product of two polynomials with integer coefficients. Explain why this is so.

Answers

a.(1) When both r and s are odd integers, the quadratic polynomial obtained by multiplying out (x - r)(x - s) will have a coefficient of 1 for x^2 term, and both the coefficient of x term and constant term will be odd integers.

(2) When both r and s are even integers, the polynomial obtained by multiplying out (x - r)(x - s) will also have a coefficient of 1 for x^2 term, but the coefficient of x term and constant term will be even integers.

(3) When one of r and s is even and the other is odd, the polynomial obtained by multiplying out (x - r)(x - s) will have a coefficient of 1 for x^2 term, the coefficient of x term will be an odd integer, while the constant term will be an even integer.

b. x^2 - 1253x + 255 cannot be written as a product of two polynomials with integer coefficients.

a. When both r and s are odd integers, the product (x − r)(x − s) will have a coefficient of 1 for x^2 term, and both the coefficient of x term and constant term will be odd integers. This is because the sum of two odd integers and the product of two odd integers is also an odd integer.

When both r and s are even integers, the product (x − r)(x − s) will also have a coefficient of 1 for x^2 term, but the coefficient of x term and constant term will be even integers. This is because the sum of two even integers and the product of two even integers is also an even integer.

When one of r and s is even and the other is odd, the product (x − r)(x − s) will have a coefficient of 1 for x^2 term, and the coefficient of x term will be an odd integer, while the constant term will be an even integer. This is because the sum of an odd and even integer is an odd integer, and the product of an odd and even integer is an even integer.

b. If x^2 - 1253x + 255 can be written as a product of two polynomials with integer coefficients, then we can write it as (x - r)(x - s) where r and s are integers. From part (a), we know that both r and s cannot be odd integers since the coefficient of x term would be odd, but 1253 is an odd integer. Similarly, both r and s cannot be even integers since the constant term would be even, but 255 is an odd integer. Therefore, one of r and s must be odd and the other must be even. However, the difference between an odd integer and an even integer is always odd, so the coefficient of x term in the product (x - r)(x - s) would be odd, which is not equal to the coefficient of x term in x^2 - 1253x + 255. Hence, x^2 - 1253x + 255 cannot be written as a product of two polynomials with integer coefficients.

Learn more about quadratic polynomial here

brainly.com/question/28988445

#SPJ4

Select which function f has an inverse g that satisfies g prime of 2 equals 1 over 6 period
f(x) = 2x3
f of x equals 1 over 8 times x cubed
f(x) = x3
1 over 3 times x cubed

Answers

The function that satisfies  F Has An Inverse G That Satisfies G'(2) = 1/6 is f(x) = 2x³ (option a).

More precisely, if f(x) is a function, then its inverse function g(x) satisfies the following two conditions:

g(f(x)) = x for all x in the domain of f

f(g(x)) = x for all x in the domain of g

In other words, if we apply f(x) to an input value x, and then apply g(x) to the resulting output, we get back to the original input value.

Now, let's look at the given condition: G'(2) = 1/6. This means that the derivative of the inverse function at x=2 is 1/6. We can use this condition to eliminate some of the options.

f(x) = 2x³

If we take the derivative of f(x), we get: f'(x) = 6x²

To find the inverse function, we can solve for x in the equation y = 2x³:

x = [tex]y/2^{(1/3)}[/tex]

Now we can express the inverse function g(x) in terms of y:

g(y) = [tex]y/2^{(1/3)}[/tex]

To find the derivative of g(x), we use the chain rule:

g'(x) = f'(g(x))⁻¹

g'(2) = f'(g(2))⁻¹

g'(2) = f'([tex]1/2^{(1/3)}[/tex])⁻¹

g'(2) = 6([tex]1/2^{(1/3)}[/tex])²)⁻¹

g'(2) = 6/36 = 1/6

Since g'(2) = 1/6, option a) is the correct answer.

To know more about function here

https://brainly.com/question/28193995

#SPJ4

Use the rational zeros theorem to find all the real zeros of the polynomial function. Use the zeros to factor f over the real numbers. f(x) = x^3 - x^2 - 37x - 35 Find the real zeros of f. Select the correct choice below and; if necessary, fill in the answer box to complete your answer. (Simplify your answer. Type an exact answer, using radicals as needed. Use integers or fractions for any rational numbers in the expression. Use a comma to separate answers as needed.) There are no real zeros. Use the real zeros to factor f. f(x)= (Simplify your answer. Type your answer in factored form. Type an exact answer, using radicals as needed. Use integers or fractions for any rational numbers in the expression.)

Answers

By using rational zeros theorem, we find that there are no real zeros of the polynomial function f(x) = x^3 - x^2 - 37x - 35, so we cannot factor f(x) over the real numbers.

To find the real zeros of the polynomial function f(x) = x^3 - x^2 - 37x - 35, we can use the rational zeros theorem, which states that any rational zeros of the function must have the form p/q, where p is a factor of the constant term (-35) and q is a factor of the leading coefficient (1).

The possible rational zeros of f are therefore ±1, ±5, ±7, ±35. We can then test each of these values using synthetic division or long division to see if they are zeros of the function. After testing all of the possible rational zeros, we find that none of them are actually zeros of the function.

Therefore, we can conclude that there are no real zeros of the function f(x) = x^3 - x^2 - 37x - 35.

However, we could factor it into linear and quadratic factors with complex coefficients using the complex zeros of f(x). But since the problem only asks for factoring over the real numbers, we can conclude that the factored form of f(x) is:

f(x) = x^3 - x^2 - 37x - 35 (cannot be factored over the real numbers)

To know more about rational zeros theorem:

https://brainly.com/question/29004642

#SPJ4

Other Questions
In applying behavior modification, a manager would take all except which of the following steps? A. pinpointing specific behaviors B. investigating the psychological (internal) causes of the behaviors C. formulating the baseline point for critical behaviors D. performing the Reward-Response analysis E. developing a plan for positive, negative, extinction and punishment reinforcements in what type of axon does saltatory conduction occur the _____ process draws upon logic, imagination, intuition, and systemic reasoning to explore possibilities and created desired outcomes. Arrange the steps to find an inverse of a modulo m for each of the following pairs of relatively prime integers using the Euclidean algorithm in the order. a = 2, m=17 Rank the options below. The ged in terms of 2 and 17 is written as 1 = 17-8.2. By using the Euclidean algorithm, 17 = 8.2 +1. The coefficient of 2 is same as 9 modulo 17. 9 is an inverse of 2 modulo 17. The Bzout coefficients of 17 and 2 are 1 and 8, respectively. a = 34, m= 89 Rank the options below. The steps to find ged(34,89) = 1 using the Euclidean algorithm is as follows. 89 = 2.34 + 21 34 = 21 + 13 21 = 13 + 8 13 = 8 + 5 8 = 5 + 3 5 = 3 + 2 3 = 2+1 Let 34s + 890= 1, where sis the inverse of 34 modulo 89. $=-34, so an inverse of 34 modulo 89 is -34, which can also be written as 55. The ged in terms of 34 and 89 is written as 1 = 3 - 2 = 3-(5-3) = 2.3-5 = 2. (8-5)- 5 = 2.8-3.5 = 2.8-3. (13-8)= 5.8-3.13 = 5. (21-13)-3.13 = 5.21-8. 13 = 5.21-8. (34-21) = 1321-8.34 = 13. (89-2.34) - 8.34 = 13.89-34. 34 a = 200, m= 1001 Rank the options below. By using the Euclidean algorithm, 1001 = 5.200 +1. Let 200s + 1001t= 1, where sis an inverse of 200 modulo 1001. The ged in terms of 1001 and 200 is written as 1 = 1001 - 5.200. s=-5, so an inverse of 200 modulo 1001 is -5. listen to the following selection from bach's cantata no. 140. what earlier composer pioneered this rhythmic technique, and for what purpose?a.Vivaldi-to teach his students how to perform rhythmic variationsb.Lully-to signify the presence of royalty in the audiencec.Corelli-to provide opportunities for imitative counterpointd.Handel-to alert the singers to an upcoming entrance b) what is the probability that an average of 22 shields will absorb more than 17.1 j/kg? use 4 decimal places. a student who reaches frustration on the oral reading passages of an informal reading inventory group of answer choices a.would usually be given a listening capacity test. b.would probably be asked to read the rest of the test silently. c.would be asked to read the word list again. d.would be assisted with all unknown words throughout the reading of the passage. HELPPPP HURRY PLSS.. Q4 NEED HELP PLEASE HELP pls answert this within 1 hour or else i will be DOOMED which of the following is not one of the necessary factors that leads to a crime according to routine activities theory? a new project has an initial cost of $165,000. the equipment will be depreciated on a straight-line basis to a zero book value over the five-year life of the project. the projected net income each year is $12,400, $16,300, $18,200, $14,300, and $10,200, respectively. what is the average accounting return? Miller, Inc., analyzed its accounts receivable balances at December 31 and arrived at the aged balances listed below, along with the percentage that is estimated to be uncollectible:Age GroupBalanceProbability ofNoncollection030 days past due$180,0001%3160 days past due40,0003%61120 days past due22,0005%121180 days past due14,00010%Over 180 days past due8,00025%=$264,000The company handles credit losses using the allowance method. The credit balance of the Allowance for Doubtful Accounts is $1,150 on December 31, before any adjustments.a. Prepare the adjusting entry for estimated credit losses on December 31.b. Prepare the journal entry to write off the Lyons Companys account on April 10 of the following year in the amount of $525. based on the information in the table, which of the following arranges the bonds in order of decreasing polarity Below is the company data for Apple Inc, currently being traded on the US markets. The measures are stated om millions of USD currency. Cash & marketable securities $165 Fixed assets $286 Net sales $1320 Earnings Before Interests and Taxes (EBIT) $143 Net Earnings After Taxes (NEAT ) $ 66 Quick Ratio ((CA-Inventory)/CL) 2.1 to 1 Current Ratio (CA/CL) 3.4 to 1 Average Collection Period (ACP) 45.60 days Return on Equity (ROE, NEAT/Net Common Equity) 13% Tax rate 25% For Apples Liabilities & Equity side, they only report common equity, debt and current operating liabilities.Based on the detailed information above, find the following calculations; (1) Accounts Receivables, (2) Current Operating Liabilities, (3) Current Assets, (4) Total Assets, (5) Net Common Equity, and (6) Debt. (10 marks) With the increase in online commerce, assume Apple has decreased its ACP by 15.60 days (i.e., totaling 30 days) while holding all other variables constant, how much cash could they generate? (10 marks). What is Apples ROIC (post-tax) ? (10 marks) what method of financing do entrepreneurs often use when they are first developing their business idea? Solve the inequality for u.48 what state of matter is rutherfordium in while at room temperature I need help! I need the graph drawn and the steps to how I got the answer but I dont know it! Please help me! assume you are preparing the customer satisfaction questionnaire. what types of questions would you include? prepare five questions that you would ask.